When is hyponormality for 2-variable weighted shifts invariant under powers?

Mathematics – Functional Analysis

Scientific paper

Rate now

  [ 0.00 ] – not rated yet Voters 0   Comments 0

Details

Scientific paper

For 2-variable weighted shifts W_{(\alpha,\beta)}(T_1, T_2) we study the invariance of (joint) k- hyponormality under the action (h,\ell) -> W_{(\alpha,\beta)}^{(h,\ell)}(T_1, T_2):=(T_1^k,T_2^{\ell}) (h,\ell >=1). We show that for every k >= 1 there exists W_{(\alpha,\beta)}(T_1, T_2) such that W_{(\alpha,\beta)}^{(h,\ell)}(T_1, T_2) is k-hyponormal (all h>=2,\ell>=1) but W_{(\alpha,\beta)}(T_1, T_2) is not k-hyponormal. On the positive side, for a class of 2-variable weighted shifts with tensor core we find a computable necessary condition for invariance. Next, we exhibit a large nontrivial class for which hyponormality is indeed invariant under all powers; moreover, for this class 2-hyponormality automatically implies subnormality. Our results partially depend on new formulas for the determinant of generalized Hilbert matrices and on criteria for their positive semi-definiteness.

No associations

LandOfFree

Say what you really think

Search LandOfFree.com for scientists and scientific papers. Rate them and share your experience with other people.

Rating

When is hyponormality for 2-variable weighted shifts invariant under powers? does not yet have a rating. At this time, there are no reviews or comments for this scientific paper.

If you have personal experience with When is hyponormality for 2-variable weighted shifts invariant under powers?, we encourage you to share that experience with our LandOfFree.com community. Your opinion is very important and When is hyponormality for 2-variable weighted shifts invariant under powers? will most certainly appreciate the feedback.

Rate now

     

Profile ID: LFWR-SCP-O-182065

  Search
All data on this website is collected from public sources. Our data reflects the most accurate information available at the time of publication.